Derivación del momento en QFT - de Energy-Momentum Tensor [cerrado]

El operador de 4 momentos conservados para el campo escalar complejo ψ = 1 2 ( ψ 1 + i ψ 2 ) se da en términos de los operadores de modo en ψ y ψ como

PAG v = d 3 pag ( 2 π ) 3 1 2 ω ( pag ) pag v ( a ( pag ) a ( pag ) + b ( pag ) b ( pag ) )
Esto solo se indica en mis notas, pero me gustaría ver cómo llegar a él usando los operadores de modo. El lagrangiano para el campo escalar complejo es
L = m ψ m ψ metro 2 ψ ψ .
El tensor de energía de tensión asociado con esta teoría es
T m v = L ( m ψ ) v ψ + v ψ L ( m ψ ) L d m v ,
que usando el lagrangiano da
T m v = m ψ v ψ + v ψ m ψ L d m v
Entonces
PAG v = T 0 v d 3 X = ( 0 ψ v ψ + v ψ 0 ψ L d 0 v ) d 3 X
entonces
PAG 0 = ( 0 ψ 0 ψ + 0 ψ 0 ψ 0 ψ 0 ψ i ψ i ψ + metro 2 ψ ψ ) d 3 X
Del mismo modo, obtengo
PAG i = d 3 X ( 0 ψ i ψ + i ψ 0 ψ )

Entiendo cómo la expresión para PAG 0 se deriva usando la integral que he escrito arriba pero la expresión para PAG i es incorrecto por un signo. Veo en mis notas que de hecho tienen la expresión integral para PAG i que tengo pero un menos en el frente. Pero no estoy seguro de la fuente de este menos. Quizás me estoy perdiendo algo conceptualmente en la derivación de PAG i entonces. Gracias por cualquier comentario.

Estoy haciendo el mismo problema: El metro 2 término simplemente no irá a ninguna parte para mí. Primero pensé que tal vez era porque la métrica, gramo 0 v = ( 1 , 0 , 0 , 0 ) y esta parte no tiene dependencia del tiempo. Sin embargo, eso no funcionaría para el hamiltoniano que contiene el metro término cuando se usa T 00 ...
Es una derivación un poco tediosa que se vuelve un poco más fácil cuando te das cuenta PAG 0 es solo el hamiltoniano, pero aún así, es solo tener cuidado, especialmente con la forma en que los derivados actúan en la expansión de mods y recordar qué ω ( pag ) es. Se hace bastante explícitamente en el capítulo 1.5 de estas notas .
En tu cálculo, ¿qué sucede con los términos sin metro 2 ? Si los incluye, debería poder factorizar un ω ( pag ) 2 = ( metro 2 + pag 2 ) del integrando, que cancela el ω ( pag ) 2 en el denominador. (Tenga en cuenta que pag 0 / ω ( pag ) = 1 .)
@Noiralef: Sí, exactamente, utilizo la relación de impulso de energía para escribir pag 0 2 pag i 2 = metro 2 lo que me permite combinar los términos que no dependían de m. Básicamente tengo
pag 0 2 pag i 2 w ( pag ) 2 + metro 2 w ( pag ) 2 = 2 metro 2 w ( pag ) 2
como está escrito en la última ecuación en mi publicación. Aquí es donde estoy atascado.
Se puede hacer mucho más simple si considera el impulso en términos de los operadores numéricos y trabaja a partir de ahí, lo que creo que es satisfactorio para su pregunta.
@CAF Creo que he resuelto tu problema. También agregué un punto adicional ya que, dado el tiempo, sospecho que estamos haciendo la misma hoja de problemas...
Para aquellos que decidieron cerrar mi hilo, consideren la edición que he hecho ahora para reabrirlo.

Respuestas (2)

Inserción de la expansión

ψ = d 3 pag ( 2 π ) 3 2 ω pag ( a pag mi i pag X + b pag mi i pag X )
en la expresión del hamiltoniano
H = d 3 X ( ψ ˙ ψ ˙ + ψ ψ + metro 2 ψ ψ )
obtenemos
H = d 3 X d 3 pag ( 2 π ) 3 2 ω pag d 3 pag ( 2 π ) 3 2 ω pag ( A + B + C )
dónde
A = ω pag ω pag ( a pag mi i pag X b pag mi i pag X ) ( a pag mi i pag X b pag mi i pag X ) B = pag pag ( a pag mi i pag X b pag mi i pag X ) ( a pag mi i pag X b pag mi i pag X ) C = metro 2 ( a pag mi i pag X + b pag mi i pag X ) ( a pag mi i pag X + b pag mi i pag X )
La integración terminada X produce dos tipos de combinaciones: a pag a pag ( 2 π ) 3 d ( pag pag ) , b pag b pag ( 2 π ) 3 d ( pag pag ) y a pag b pag ( 2 π ) 3 d ( pag + pag ) , b pag a pag ( 2 π ) 3 d ( pag + pag ) . El valor esperado de estos últimos en cualquier estado propio de cantidad de movimiento es obviamente cero, por lo que no contribuyen al hamiltoniano. integrando sobre pag y usando la relación ω pag 2 = | pag | 2 + metro 2 Nosotros recibiremos
H = d 3 pag ( 2 π ) 3 2 ω pag ω pag ( a pag a pag + b pag b pag )

Gracias por tu respuesta. ¿Podría explicar por qué 'los valores esperados de los últimos en cualquier estado propio de impulso son obviamente cero'? ¿Es porque esos términos vienen con un exponencial oscilante y por lo tanto se desvanecen sobre todo p?
@CAF Por ejemplo, a pag b pag crea una partícula con momento pag y una antipartícula con impulso pag , por lo tanto, aplicarlo a un estado propio de impulso produce otro estado propio que difiere del original, por lo que su producto interno es cero.
Entonces, ¿quieres decir eso? a pag b pag | 0 = | tu pag , v ¯ pag ? Pero, ¿de qué manera diferirá el estado propio si aplicamos este operador al estado nuevamente? ¿No producirá simplemente un estado con dos partículas de impulso? pag y dos antipartículas de cantidad de movimiento pag ?
Los estados propios con la misma cantidad de movimiento pero diferente número de partículas son vectores propios diferentes de norte , el operador de número de partículas, por lo que son ortogonales.
Ah ya veo gracias! Entonces, a partir de este argumento, a pag a pag | tu pag , tu pag es un estado con 2 partículas de impulso pag . Desde | tu pag , tu pag es también un estado con 2 partículas, son los mismos vectores propios de N y por lo tanto no desaparecen. Así que los términos restantes deben ser simplemente a a y b b . ¿Es eso correcto?
@CAF No exactamente. Dejar | α Sea cualquier estado propio de cantidad de movimiento y número de partículas, entonces α | a pag a pag | α desaparece a menos que pag = pag . Aquí tenemos d ( pag pag ) , así queda; pero α | a pag b pag | α siempre se desvanecerá. Debido a que la matriz de la suma de esos términos sin una creación y un operador de aniquilación en pares es diagonal, también se desvanece.
¡Gracias! Lamento ser una molestia, pero tal vez podría editar su respuesta para incluir también el punto de partida de la informática. PAG i ? Como escribí en mi publicación de apertura, tengo que
PAG i = d 3 X T 0 i = ( 0 ψ i ψ + i ψ 0 ψ ) d 3 X
Al realizar este cálculo, obtengo menos cuál debería ser la respuesta. Este inconveniente es que simplemente no sé cómo deshacerme de él y he estado atrapado con él durante tres días. ¡Gracias!
@CAF En la métrica ( 1 , 1 , 1 , 1 ) , i = X i , creo que es donde radica tu error. Si no, incluiré el cálculo del impulso espacial.
Pero, ¿por qué tendría que trabajar con X i ? acabo de trabajar con i y al tomar derivados, usó el hecho de que
± i i ( pag X ) = ± i i ( pag o t pag j X j ) = i pag i .
Continuando con esto, obtengo el error negativo al final. ¡Gracias!
@C Y F m Se define como m = X m , y para i = 1 , 2 , 3 , i = X i = X i , de este modo i ( pag j X j ) = pag i . Cometes un error con el letrero aquí.

Para el momento complejo,

T m v = m ϕ ( X ) v ϕ ( X ) + v ϕ ( X ) m ϕ ( X ) gramo v m L
Ahora puedes considerar dos casos separados:

  1. T 0 i que da el impulso 3, PAG i es decir gramo i 0 = ( 0 , 0 , 0 )
  2. T 00 que da el hamiltoniano, H = PAG 0 es decir gramo i 0 = 1 (depende de la métrica puede ser -1)

trabajar desde aquí es sencillo y este método se cita en:

  • Greiner, pág. 82
  • Weinberg, pág. 310

Si también sugeriría encarecidamente mirar la página 286 de Schwabl, ya que tiene un resultado interesante que involucra,

PAG = pag pag ( norte ^ a (   metro a t h b F pag ) + norte ^ b (   metro a t h b F pag ) )

Gracias por su respuesta. Dos cosas que no entiendo: es obvio (por inspección) que la densidad lagrangiana se desvanece para configuraciones de campo que resuelven la ecuación de Dirac, pero esto no es tan claro para el lagrangiano para un campo escalar complejo. Además, por lo que puedo ver, L para un campo escalar complejo contiene un término de la forma m ψ (Como debe ser de otra manera L no es real). Por favor, dime si estás de acuerdo con esto.
De hecho, mi profesor me dijo que tenía algo que ver con el gramo i 0 en realidad es una suma sobre el campo 3 en lugar del campo 4, por lo tanto, solo contiene 0 . Parece ser una de esas derivaciones en las que cada libro y recurso usa la molesta frase "es claro para ver" cuando en realidad no es tan claro para un estudiante que está aprendiendo el tema. Sin embargo, fundamentalmente, puede saber con seguridad que desaparece para terminar su prueba. Intentaré obtener más detalles sobre exactamente por qué durante el próximo día más o menos.
Sí estoy de acuerdo con usted para la evaluación de PAG i . En ese caso hay un componente gramo 0 i multiplicando L así que de hecho se desvanece. Sin embargo, en el caso de PAG 0 esto no es cierto y es el caso con el que tengo dificultades. Aunque lo derivé usando la observación de ACuriousMind, pero todavía me gustaría ver por qué mi método no me está dando lo que quiero.
@CAF Tengo la respuesta formal: gramo 0 v se puede pensar en dos casos separados 1) gramo 0 i = ( 0 , 0 , 0 ) y 2) gramo 0 0 = 1 (o -1 dependiendo de su definición de métrica). 1) le dará la definición de PAG m y 2) le dará la definición de H = PAG 0 . Esta es precisamente la razón por la cual las dos definiciones siempre están explícitamente separadas en toda la literatura, de lo contrario, solo podría combinarlas como una sola.
Solo me preguntaba, ¿qué expresión inicial usaste para calcular PAG i ? Yo tenía
PAG i = ( o ψ i ψ + i ψ o ψ ) d 3 X
que se sigue exactamente de la definición del tensor de energía de tensión pero no da la respuesta correcta. Pensé que tenía PAG i pero noté un error de señal en lo que tenía anteriormente.
Encontré un error dudoso que estoy corrigiendo yo mismo... ver editar